If y varies directly with x, and y = 12 when x = 8, write the direct linear variationequation.O y=8xO y = 12xO y=2/3xO y= 3/2 x

Answers

Answer 1

A direct linear variation of y with x has the general form:

[tex]y=mx[/tex]

Where m is the ratio of y to x, which is a y divided by x.

Since we know that when x equals 8, y equals 12, we can calculate m, like this:

[tex]m=\frac{y}{x}=\frac{12}{8}=\frac{6}{4}=\frac{3}{2}[/tex]

Now that we know that m=3/2, the linear variation equation would be:

[tex]y=\frac{3}{2}x[/tex]


Related Questions

A job placement agency advertised that last year its clients, on average, had a starting salary of $39,500. Assuming that average refers to the mean, which of the following claims must be true based on this information?Note: More than one statement could be true. If none of the statements is true, mark the appropriate box.Last year some of their clients had a starting salary of at least $39,500 .Two years ago some of their clients had a starting salary of at least $39,500 .Last year, the number of their clients who had a starting salary of more than $39,500 was equal to the number of their clients who had a starting salary of less than $39,500.Last year at least one of their clients had a starting salary of more than $42,000.Last year at least one of their clients had a starting salary of exactly $39,500.None of the above statements are true.

Answers

In the question, it is given that the average salary is $39,500.

In consideration of the first statement

(a) , last year, some of their clients had a starting of atleast $39500 ...this is true

(b) they have mentioned the case of last two years, this is also incorrect.

( c )if the client has lesser than $39,500 salary, and the average salary is $39,500 , then average will be less than $39,500, then statement a is not true..

(d) Last year at least one of their clients had a starting salary of more than $42,000., this is more than the average , but could be true, but it is false as $42000 will be more than the average of $39,500

(e) Last year at least one of their clients had a starting salary of exactly $39,500. ... this is not true, as exactly would not allow the $39,500 to be any less.

• So correct options would be A

Don'te is sitting on the bus on the way home from school and is thinking about the fact that he has three homeworkassignments to do tonight. The table below shows his estimated probabilities of completing 0, 1, 2, or all 3 of theassignments.Number of Homework Assignments Completed0123Probability162951813What is the probability he will not do exactly 1 assignment?2/9Ob 7/9Ос 7/18Od 1

Answers

SOLUTION

Don'te is sitting on the bus on the way home from school and is thinking about the fact that he has three homework assignments to do tonight.

The table below shows his estimated probabilities of completing 0, 1, 2, or all 3 of the

assignments.

Number of Homework

Assignments Completed Probability

0 1/ 6

1 2/ 9

2 5/18

3 1/ 3

What is the probability he will NOT do exactly 1 assignment?

1/ 6 + 5 / 18 + 1/ 3 = 7 / 9 ................. OPTION B

4(−5x − 6) = 4(9x + 4)

Answers

Answer: X= -5/7

Step-by-step explanation:

4(-5x-6)=4(9x+4)

-20x-24=4(9x+4)

-20x-24=4(9x+4)

-20x-24=36x+16

Then add 24 to both sides:

Assume the radius of a certain planet is 2460 km and the planet is a sphere. What is its surface area?

Answers

Answer:

Explanation:

The surface area of a sphere is calculated using the formula:

[tex]A=4\pi r^2[/tex]

Given that the radius of a certain spherical planet, r = 2460 km

[tex]undefined[/tex]

f(x) = 5x2 – 7(4x + 3). What is the value of f(3)?

Answers

Given the function:

[tex]f(x)=5x^2-7(4x+3)[/tex]

we can find f(3) by making x = 3 and solving for the term f. In this case, we have the following:

[tex]\begin{gathered} f(3)=5(3)^2-7(4(3)+3) \\ =5(9)-7(12+3)=45-7(15)=45-105=-60 \\ \Rightarrow f(3)=-60 \end{gathered}[/tex]

therefore, f(3) = -60

19. We saw 10 ladybugs. We saw 5 butterflies. How many more ladybugs than butterflies did we see?

Answers

Given that,

Total ladybugs = 10

Total butterflies = 5

More ladybugs than butterflies are = ladybugs - butterflies

=> 10 - 5

=> 5

Therefore, there will be 5 more ladybugs than butterflies.

I need help with this, i dont know what to do

Answers

We have given that

[tex]PQ=ST[/tex][tex]QR=TR[/tex]

Given that R is the midpoint so

[tex]PR=SR[/tex]

Hence

[tex]\Delta PQR\cong\Delta STR[/tex]

BY SSS

The quotient of 93 and x

Answers

The quotient of ;

[tex]undefined[/tex]

I need help with a homework

Answers

Consider the triangle PAM and triangle PBM.

[tex]\begin{gathered} \angle PMA=\angle PMB\text{ (Each angle is right angle)} \\ AM=BM\text{ (M is perpendicular bisector of AB)} \\ PM\cong PM\text{ (Common side)} \\ \Delta\text{PMA}\cong\Delta\text{PMB (By SAS similarity)} \\ PA\cong PB\text{ (Corresponding part of Congurent triangle)} \end{gathered}[/tex]

Hence it is proved that,

[tex]PA\cong PB[/tex]

My questions are: #1) Determine the account balance at 4 years if $20,000 was invested in an account that compounds daily at 4.5% per year.#2) Determine the account balance at 5 years if $20,000 was invested in an account that compounds continuously at 4.5% per year.#3) A bacterial culture grows from 10 bacteria at 1.5% per minute starting at 7:00 a.m. find bacteria count after 12 hours if continues growth is assumed. (round down to the nearest whole bacterium)

Answers

We have the following formula:

[tex]P(t)=10\cdot(1.015)^t[/tex]

where t is the amount of minutes we have waited. So in this case we have 12hours, therefore we have waited 12*60=720 minutes

so we have that after 720 minutes the population of bacteria is

[tex]10\cdot(1.015)^{720}=452428.98\approx452429[/tex]

so the answer is 452429

For what value of x does f(x) = 1?
Answer Choices:
A. x = 0
B. x= 1
C. x = 5
D. x = -5

Answers

B pretty sure it would be b

Suppose you ride your bicycle to the library traveling at .50 km/min. It takes you 25minutes to get to the library. How far did you travel

Answers

Answer:

50km

Step-by-step explanation:

We know that every minute we travel .5 km....  

[tex]\frac{1 min}{.50 km}[/tex]

But we want to know how far we traveled in 25 mins

[tex]\frac{25 mins}{? km}[/tex]    

So we go and do....

[tex]\frac{1 min}{.50 km} * \frac{25 mins}{? km}[/tex]

Then we have to divide 25 by .50

[tex]\frac{25 mins}{.50 km}[/tex]

Which Gives us

50 so he traveled 50km

Answer:

12.5km

Step-by-step explanation:

Hey! Let's help you with your question here!

We can begin by figuring out what we know!

Known InformationBicycle traveling at .50km/min (0.50km/min)It takes 25 minutes to get to the library.

What we don't know and solving for it

What we don't know is the distance traveled based on the given time. Now, what do we do? Well, we already know that the bicycle is traveling at a distance of .50km per minute and it takes us 25 minutes to get to the library. All we need to do here is take the distance per minute and multiply it by the total amount of minutes it takes to reach the destination. It would look something like this:

[tex]=(0.50km/s)*25[/tex]

[tex]=0.50*25[/tex]

[tex]=12.5[/tex]

Therefore, we can see that if we travel at a distance of .50km/min for 25 total minutes, we get a final distance of 12.5km.

Find the area of quadrilateral math with vertices M(7, 6), A(3, - 2), T(- 7, 1) and H(- 1, 9)

Answers

Lets draw a picture of our quadrilateral:

In order to find the area, we can divide our parallelogram in 2 triangles:

The area of triangle AHT is given by

[tex]\text{Area }\Delta AHT=\frac{1}{2}(x_1(y_2-y_3)+x_2(y_3-y_1)+x_3(y_1-y_2))[/tex]

where

[tex]\begin{gathered} (x_1,y_1)=(3,-2)=A \\ (x_2,y_2)=(-1,9)=H \\ (x_3,y_3)=(-7,1)=T \end{gathered}[/tex]

By substituting these points into the given formula, we get

[tex]\text{Area }\Delta AHT=\frac{1}{2}(3_{}(9_{}-(-7))-1((-7)-(-2))-7((-2)-9))[/tex]

which gives

[tex]\begin{gathered} \text{Area }\Delta AHT=\frac{1}{2}(3_{}(16)-1(-5)-7(-11)) \\ \text{Area }\Delta AHT=\frac{1}{2}(48+5+77) \\ \text{Area }\Delta AHT=\frac{130}{2} \\ \text{Area }\Delta AHT=65 \end{gathered}[/tex]

Similarly, for the area of triangle AHM, we can choose

[tex]\begin{gathered} (x_1,y_1)=(3,-2)=A \\ (x_2,y_2)=(-1,9)=H \\ (x_3,y_3)=(7,6)=M \end{gathered}[/tex]

By substuting in our area formula, we get

[tex]\text{Area }\Delta AHM=\frac{1}{2}(3_{}(9_{}-6)-1(6-(-2))+7((-2)-9))[/tex]

which gives

[tex]\begin{gathered} \text{Area }\Delta AHM=\frac{1}{2}(3_{}(3)-1(8)+7(-11) \\ \text{Area }\Delta AHM=\frac{1}{2}(9-8-77) \\ \text{Area }\Delta AHM=\frac{76}{2} \\ \text{Area }\Delta AHM=38 \end{gathered}[/tex]

Then, the total area is given by

[tex]\begin{gathered} A=\text{Area }\Delta AHT+\text{Area }\Delta\text{AHM} \\ A=65+38 \\ A=103 \end{gathered}[/tex]

then, the answer is 103 units squared.

Points EGNK or midpoints explain how you know figure EGHK is a parallelogram

Answers

Solution

Given a triangle FDH with E, G and H the midpoints of the sides of the triangle

Considering the figure EGHK,

Line EG is parallel and equal to line KH

Line, EK is parallel and equal to line GH

This is a feature of a parallelogram i.e two pair of opposite sides of a parallelogram are parallel and equal

∠GEK is equal to ∠GHK and ∠EGH is equal to ∠EKH

This is a feature of a parallelogram i.e

English Do the head bean to see how many Ms Elkot has gallons of gas in her, and the car uses 1 of a gallon of gas on the drive to work How can Ms Emo Egure out how many trips to work she can make? Check all that apply use the expression 6/8 / 1/4 to find the answer 3 orange parts fit on the blue parts 2 blue parts fit on the orange part Ms Elliot can make 2 trips to school Ms Ellot can make 3 trips to school

Answers

Since she needs 1/4 of gallons and she has 6/8 gallons, then she can use the expression

[tex]\frac{6}{8}\text{ \%}\frac{1}{4}[/tex]

to find out haw many trips she can make

Leah's Cafe has regular coffee and decaffeinated coffee. This morning, the cafe served 5 coffees in all, 20% of which were regular. How many regular coffees did the cafe serve? regular coffees

Answers

Let regular coffee be x and decaffeinated be y. If they served 5 coffees in all, and

Function gis represented by the equation.915) = –18(3) *+ 2Which statement correctly compares the two functions on the interval [-1, 2]?

Answers

step 1

Find out the average rate of change function f over the interval [-1,2]

[tex]\frac{f(b)-f(a)}{b-a}[/tex]

we have

a=-1

b=2

f(a)=f(-1)=-22

f(b)=f(2)=-1

substitute

[tex]\frac{-1-(-22)}{2-(-1)}=\frac{21}{3}=7[/tex]

step 2

Find out the average rate of change function g(x) over the interval [-1,2]

we have

a=-1

b=2

g(a)=g(-1)=-18(1/3)^-1+2=-52

g(b)=g(2)=-18(1/3)^2+2=0

substitute

[tex]\frac{0-(-52)}{2-(-1)}=\frac{52}{3}=17.3[/tex]

therefore

17>7

the answer is option A

[tex]x + y = - 2 \\ 3x - y = - 2[/tex]draw each line and estimate the solution.

Answers

We can draw each line by assuming that x = 0 and y = 0 and solve each case.

In the first equation, we have the following:

[tex]\begin{gathered} x+y=-2 \\ x=0\Rightarrow y=-2 \\ y=0\Rightarrow x=-2 \end{gathered}[/tex]

notice that we have a pair of coordinate points (0,-2) and (-2,0). These two points will be useful when we draw the line.

Next, for the second equation we have:

[tex]\begin{gathered} 3x-y=-2 \\ x=0\Rightarrow-y=-2\Rightarrow y=2 \\ y=0\Rightarrow3x=-2\Rightarrow x=-\frac{2}{3} \end{gathered}[/tex]

in this case we have the points (0,2) and (-2/3,0). Now, if we draw both lines on the coordinate plane we get the following:

notice that both lines intersect on the point (-1,-1). Thus, the solution of the system of equations is the point (-1,-1)

Question 3. Y=(1/5)^xSketch the graph of each of the exponential functions and label three points on each graph.

Answers

Given exponential function:

[tex]y\text{ = (}\frac{1}{5})^x[/tex]

Let us obtain three points including the y-intercept so that we can plot the function y = f(x)

When x =0:

[tex]\begin{gathered} y\text{ = (}\frac{1}{5})^0 \\ =\text{ 1} \end{gathered}[/tex]

when x =1:

[tex]\begin{gathered} y\text{ = (}\frac{1}{5})^1 \\ =\text{ }\frac{1}{5} \end{gathered}[/tex]

when x =2:

[tex]\begin{gathered} y\text{ = (}\frac{1}{5})^2 \\ =\text{ }\frac{1}{25} \end{gathered}[/tex]

We have the points : (0, 1), (1, 1/5), and (2, 1/25)

Using these points, let us provide a sketch of the plot of y =f(x). We have the plot as shown below:

write the equation of the line, with the given properties, in slope-intercept form.Slope=-6, through (-8,8)

Answers

The given slope is:

[tex]m=-6[/tex]

And the point:

[tex](-8,8)[/tex]

we label the coordinates as follows:

[tex]\begin{gathered} x_1=-8 \\ y_1=8 \end{gathered}[/tex]

And now, we use the slope-point formula, which is:

[tex]y-y_1=m(x-x_1)[/tex]

substituting the known values of slope m and the point:

[tex]y-8=-6(x-(-8))[/tex]

We need to solve this for y to find the slope intercept form (which is y=mx+b):

[tex]\begin{gathered} y-8=-6(x+8) \\ y-8=-6x-48 \\ y=-6x-48+8 \\ y=-6x-40 \end{gathered}[/tex]

The slope-intercept form is:

y = -6x - 40

The cost of a pound of nails increased from $2.03 to $2.19. What is the percent of increase to the nearest whole-number percent?(Type an integer

Answers

Hello there. To solve this question, we'll have to remember some properties about percents.

We start with percent of increase: it is the difference between how much a thing is from another and 100%.

Now, to calculate this amount, we take the ratio of the numbers. In this case, the cost of a pound of nails.

Knowing it increased from $2.03 to $2.19, we calculate:

2.19/2.03 = 1.079

Multiply by 100% to find its amount in percent

1.079 * 100% = 107.9%

Now, we simply take the difference:

107.9% - 100% = 7.9%

Rounding this percent to the nearest whole-number percent, we get:

8%

28 Solve. 15 = 4n - 5

Answers

We have the next equation

[tex]15=4n-5[/tex]

then we need to isolate the n

[tex]\begin{gathered} 4n=15+5 \\ 4n=20 \\ n=\frac{20}{4} \\ n=5 \end{gathered}[/tex]

the value of n is 5

Brennan puts 600.00 into an account to use for school expenses the account earns 11%interest compounded annually how much will be in the account after 6 years

Answers

Here,

P = 600

t = 6

n = 1 (annually)

r = 11% = 0.11

Applying the fromula to calculate compound interest we have,

[tex]\begin{gathered} A=P(1+\frac{r}{n})^{nt} \\ \text{ =600(1+0.11)}^6 \\ \text{ =1122.248} \end{gathered}[/tex]

The answer is 1122.248.

P= rt , Solve for t in this literal equation

Answers

Answer:

t = P/r

Explanations:

The given equation is:

P = rt

To solve for t by dividing both sides by r

[tex]\begin{gathered} \frac{P}{r}=\frac{rt}{r} \\ \frac{P}{r}=t \end{gathered}[/tex]

Therefore:

[tex]t\text{ = }\frac{P}{r}[/tex]

We want to solve the following system of equations.x^2 + y^2 = 1y = 2x + 2One of the solutions to this system is (-1,0).Find the other solution.

Answers

Answer:

(-3/5, 4/5)

Explanation:

Given the system of equations:

[tex]\begin{gathered} x^2+y^2=1 \\ y=2x+2 \end{gathered}[/tex]

First, we substitute y=2x+2 into the first equation to obtain:

[tex]\begin{gathered} x^2+(2x+2)^2=1 \\ x^2+(2x+2)(2x+2)=1 \\ x^2+4x^2+4x+4x+4=1 \\ 5x^2+8x+4-1=0 \\ 5x^2+8x+3=0 \end{gathered}[/tex]

We solve the derived quadratic equation for x,

[tex]\begin{gathered} 5x^2+8x+3=0 \\ 5x^2+5x+3x+3=0 \\ 5x(x+1)+3(x+1)=0 \\ (5x+3)(x+1)=0 \\ 5x+3=0\text{ or }x+1=0 \\ x=-\frac{3}{5}\text{ or -1} \end{gathered}[/tex]

We then solve for the corresponding values of y using any of the equations.

[tex]\begin{gathered} \text{When x=-1} \\ y=2x+2 \\ y=2(-1)+2 \\ y=0 \\ When\text{ }x=-\frac{3}{5} \\ y=2(-\frac{3}{5})+2 \\ =\frac{4}{5} \end{gathered}[/tex]

Therefore, the solutions o this system are:

(-1,0) and (-3/5, 4/5).

The other solution is (-3/5, 4/5).

consider the relationship between f(x)=2^x and g(x)=log2 x.g is a reflection of f over the line y=x.True or False

Answers

the function

[tex]\log _2x[/tex]

is the inverse function of

[tex]2^x^{}[/tex]

On the graph, the inverse of a function is the reflection of the original function over the line y = x. Then, the statement is true

Can I get a walk through on how this is solved.?

Answers

Answer:

1 1/2 quarts of water

Explanation:

If she drinks 1/4 quart of water for every mile, in 6 miles, she will drink 6 times 1/4 quart of water, so

[tex]6\times\frac{1}{4}=\frac{6}{1}\times\frac{1}{4}=\frac{6\times1}{1\times4}=\frac{6}{4}[/tex]

Now, we can simplify the fraction dividing the numerator and denominator by 2

[tex]\frac{6}{4}=\frac{6\div2}{4\div2}=\frac{3}{2}[/tex]

Now to convert 3/2 to a mixed number, let's divide 3 by 2

Since 1 is the quotient and 1 is the remainder, the mixed number is

[tex]\begin{gathered} \frac{3}{2}=\text{Quotient}\frac{\text{ Remainder}}{2} \\ \frac{3}{2}=1\frac{1}{2} \end{gathered}[/tex]

So, the answer is;

1 1/2 quarts of water.

Use inductive reasoning to find the next number in the pattern: 1 / 3 , 2 / 4, 3 / 5, ____.

Answers

..

SOLUTION

[tex]\frac{1}{3},\frac{2}{4},\frac{3}{5},...[/tex]

The sequence progresses by the addition of 1 to both numberator and denominator.

[tex]\frac{3+1}{5+1}=\frac{4}{6}[/tex]

The next number is 4/6.

QuestionThe following is a data set of the average weekly number of cups of coffee consumed by employees in an office. Find the mean and median and determine if the mean or median is the better measure of central tendency.5,0,5,2,0,10,7,8,10,21,5,8,2,5,3,5Select the correct answer below:Mean = 5, Median = 6The median is the better measure of central tendency.Mean = 5, Median = 6The mean is the better measure of central tendency.Mean = 6, Median = 5The median is the better measure of central tendency.Mean = 6, Median = 5The mean is the better measure of central tendency.

Answers

Explanation

we will begin by finding the mean and median of the data set

The mean is simply the average of the set, which will be

[tex]mean=\frac{5+0+5+2+0+10+7+8+10+21+5+8+2+5+3+5}{16}=\frac{96}{16}=6[/tex]

The median is

[tex]\begin{gathered} \mathrm{The\:median\:is\:the\:value\:separating\:the\:higher\:half\:of\:the\:data\:set,\:from\:the\:lower\:half.} \\ \mathrm{If\:the\:number\:of\:terms\:is\:odd,\:then\:the\:median\:is\:the\:middle\:element\:of\:the\:sorted\:set} \\ If\:the\:number\:of\:terms\:\:is\:even,\:then\:the\:median\:is\:the\:arithmetic\:mean\:of\:the\:two\:middle\:elements\:of\:the\:sorted\:set \end{gathered}[/tex]

Thus, we have the median as 5

To check which is a better measure, we will have to check the skewness

The skew value is 1.51

This means it is positively skewed

Thus

If the distribution is positively skewed then the mean is greater than the median which is in turn greater than the mode.

Therefore, the answer is

Mikel creates the table below to help her determine 40 percent of 70

Answers

We want to determine the 40 percent of 70, so we have to multiply 70 by 40%:

[tex]\begin{gathered} 40\text{ percent=}\frac{40}{100} \\ 70\cdot40\text{ percent=70}\cdot\frac{40}{100}=\frac{2800}{100}=28 \end{gathered}[/tex]

Other Questions
Solve the system of equations x + 2y = 0 Solve the equation for x. how might the spanish armada attempt to invade england have ended if a storm had not destroyed the spanish fleet(please i need it quick!!) The internal energy of an expanding gas changes by 2000j. How much work is done if the process is done adiabatically? What is the equation of the line that passes through the point (5, 3)and has a slope of -4/5 Which definition best describes a perpendicular bisector?Required to answer. Single choice. A line segment from a vertex of the triangles to the opposite side that divides an angle into two congruent adjacent angles. A line segment from a vertex of the triangles to the midpoint of the opposite side.A line segment from any vertex perpendicular to the line containing the opposite side of a triangle.A line that is perpendicular to a side of the triangle and also bisects that side of the triangle (it goes through the midpoint). the scatter plot below was constructed using data on length and inches (x) of several alligators and weight in pounds (y)right if you sent this describing the relationship between weight and length for these alligators Five students, Stella, Victoria, Alexander, Eva, and Hunter, line up one behind theother. How many different ways can they stand in line? Which system of inequalities is shown? O Ayx 2 y2 . . y2 . 6. Analyze Marc has one dollar, one quarter, one dime, one nickel, andone penny. He spends 35 cents. How much money does he have left?A $0.76B $0.96$1.06D $1.16 The expression -7y is a ____. (20 Points!!) (Please help!)TermConstantVariable Sidney's group came up with a design for the seat belt but had to figure out whattype of material to use. They picked two fabrics: one that was very rigid with littlegive and a second fabric that was stretchy. They made the two belts, got somemotorized cars and put action figures into the cars, one car per seat belt design.Then they ran the cars into a wall of the classroom. Predict which belt they choseand why.The stretch seat belt: for every action there is a reaction. The stretch belt letsyou move forward and then you move back again.The non-stretch seat belt: with no seatbelt to stop the driver with the car, thedriver flies free until stopped suddenly by impact on the steering column so thenon-stretch belt keeps the driver from flying free.The non-stretch seat belt holds the driver immobile against the seat as the carcrashed.The stretch seat belt: some stretch in the seatbelts will reduce the averageimpact force by extending the stopping distance of the passenger the current stock price is 20 per share. the price at the end of a four-month period is modeled with a one-period binomial tree so that the stock price can either increase by $5, or decrease by $5. the stock pays dividends continuously with the dividend yield 0.04. the continuously compounded, risk-free interest rate is 0.05. what is the stock investment in a replicating portfolio for four-month, $20-strike european call option on the above stock? Mr. March sells popcorn at his theater. He uses 3 3/4 cups of unpopped corn to make 15 bags of popped corn. Write an equation for the number of bags of popcorn b that can be made with c cups of unpopped corn. A towns population increases at a constant rate. In 2010 the population was 56,000 . By 2012 the population had increased to 81,000 . If this trend continues, predict the population in 2016. The population will be Number in 2016. divide decimals by decimals 033 divided by 688 6. Examine the two-way frequency table below.Gold Medals Silver Medals Bronze MedalsUSA 201842Spain 2511France 192726Based on the data in the two-way frequency table, what is the probability that a randomly selected player won a bronze medal given that the player represented Spain?22.4%24.4%13.995.5%PREVIOURPREVIOUS6 ofNEXTREVIEWSALSion outINTL Laboratory balances that measure to the hundredths (0.01g) are calleda) centigram balanceb) milligram balancec) analytical balance hello I don't you can help me with this please A student plays the following game. He tossed three coins. If he gets exactly two heads he wins $5. If he gets exactly one head he wins $3. Otherwise, he loses $2. On the average, how much should he win or lose per play of the game? 37.In the compound MgO, what is the oxidation number of oxygen?Select one:a. +4b. +2c. +1d. -2